Einzelsequenz der Drehimpulsleiter in der Quantenmechanik? - Warum gibt es nur a

Wie beweisen Sie, dass es nur eine Folge von Drehimpuls-Eigenzuständen gibt, die durch den Leiteroperator verbunden sind, innerhalb des Unterraums, in dem der quadratische Modul des Drehimpulses einen gegebenen Eigenwert hat?

Genauer gesagt lassen J ^ sei ein (verallgemeinerter) Drehimpulsoperator, definiert als hermitescher Operator, dessen kartesische Komponenten die Kommutierungsrelationen erfüllen,

[ J ^ X , J ^ j ] = ich J ^ z , [ J ^ j , J ^ z ] = ich J ^ X , [ J ^ z , J ^ X ] = ich J ^ j .
Wir definieren die Erhöhung ( J ^ + ) und Absenken ( J ^ ) Operatoren als
J ^ + = J ^ X + ich J ^ j , J ^ = J ^ X ich J ^ j .
Lassen Sie uns einen generischen Namen, Leiteroperator, geben J ^ + Und J ^ . Nennen wir den folgenden Operator als quadratischen Modul des Drehimpulses,
J ^ 2 = J ^ X 2 + J ^ j 2 + J ^ z 2 .

Aus der Kommutierungsrelation [ J ^ 2 , J ^ X ] = 0 haben diese beiden Operatoren simultane Eigenzustände, die wir schreiben als | X a , Y β , bedeutet, dass

J ^ 2 | X a , Y β = X a | X a , Y β , J ^ z | X a , Y β = Y β | X a , Y β .
Aus den Vertauschungsbeziehungen [ J ^ 2 , J ^ ± ] = 0 Und [ J ^ z , J ^ ± ] = ± J ^ ± , es folgt dem
J ^ ± | X a , Y β >= | X a , Y β ± > C ± ( X a , Y β ) ,
Wo C ± ( X a , Y β ) ist eine komplexe Zahl. Aus der Tatsache, dass
X a , Y β | ( J ^ 2 J ^ z 2 ) | X a , Y β = X a , Y β | ( J ^ X 2 + J ^ j 2 ) | X a , Y β 0 ,
es folgt dem
X a Y β 2 0.
Daher ist der Eigenwert von J ^ z hat sowohl die untere als auch die obere Grenze, die durch den Eigenwert von bestimmt sind J ^ 2 . Soweit so gut für mich.

Alle wenigen Lehrbücher, die ich mir angesehen habe, folgen im Wesentlichen einer Art von Argument, um die Werte zu bestimmen X a Und Y β [1]. Dieses Argument basiert auf der Behauptung, dass

  • sukzessive Multiplikation von J ^ + An | X a , Y Mindest muss dazu führen | X a , Y max D , ..... (A1)

Wo Y Mindest Und Y max sind der minimale und der maximale Wert, der von übernommen wird Y β , Und D ist eine komplexe Zahl.

Dann wird daraus abgeleitet X J = J ( J + 1 ) 2 mit J = 0 , 1 / 2 , 1 , 3 / 2 , , Und Y M = M mit M = J , J + 1 , , J 1 , J , wo ich die Indizes geändert habe a Und β Zu J Und M um der konventionellen Notation zu folgen. Ich sehe jedoch keine Rechtfertigung des oben zitierten Anspruchs (A1).

Nehme an, dass 0 < A < und das Y β = Y Mindest + A ist ein Eigenwert von J ^ z . Welcher Widerspruch ergibt sich? Anwendung von J ^ + An | X a , Y Mindest + A würde nur eine andere Folge von Eigenzuständen von offenbaren J ^ z mit Eigenwerten Y Mindest + A , Y Mindest + A + , , Y Mindest + A + M , Wo M ist die ganze Zahl so dass Y max < Y Mindest + A + M Y max . Insbesondere, welcher Widerspruch entsteht, wenn im letzten Ausdruck die Gleichheit gilt, also existiert A so dass ( Y max Y Mindest A ) / eine ganze Zahl ist und somit Behauptung (A1) falsch ist?

Diese Frage,

Drehimpuls - Beweis für ganzzahlige oder halbzahlige Eigenwerte

möglicherweise beabsichtigt, dieselbe Frage zu stellen, als sie in dem (unbekannten) Lehrbuch gestellt wurde, das vom OP dieser Frage untersucht wurde, aber beide der beiden darin enthaltenen Antworten gehen von der hier betroffenen Behauptung (A1) aus, und das OP akzeptierte eine der Antworten . Deshalb stelle ich hier diese neue Frage, um den Punkt der Besorgnis zu präzisieren.

Diese Frage,

Warum machen S l ( 2 , C ) Hebe- und Senkoperatoren J ± quantisierte Eigenwerte garantieren?

fragt nach einem etwas anderen Punkt, dh der Existenz von Leiteroperatoren mit fraktionalem Einstieg .

[1] Das Argument in Lehrbüchern ist im Wesentlichen dasselbe wie in diesem Antwortbeitrag:

https://physics.stackexchange.com/a/128918/6399

Ich habe die Notation teilweise aus diesem Beitrag übernommen.

Um die Invarianz der Wellenfunktion bei einer vollständigen Drehung (von 2 π oder 4 π ) erhalten wir eine Einschränkung auf die möglichen Drehimpulszustände als ganze (oder halbe) Zahlen.

Antworten (2)

Drehimpulsoperator J ist der Generator für Drehungen auf einer Wellenfunktion. Das heißt, wenn Sie einen Staat haben | ψ und Sie möchten es in Form von Koordinaten ausdrücken, die um eine Achse gedreht sind N ^ um einen Winkel θ , dies ist gegeben durch

exp ( ich J N ^ θ ) | ψ
Jetzt Invarianz von | ψ unter Rotation von 2 N π schränkt die Eigenwerte von ein J ganzzahlige Vielfache von sein . Wir schöpfen also alle möglichen Zustände beim Zählen über Ladder-Operatoren aus.

Lassen Sie uns überlegen | ψ so dass J ^ z | ψ = ( / 2 ) | ψ . Auf 2 π Drehung um die z Achse, exp ( ich 2 π J ^ z / ) | ψ = | ψ e ich π , und es erhält den Phasenfaktor. Das | ψ ist nur auf unveränderlich 4 π Drehung. Nun nehme an | ϕ ist so das J ^ z | ϕ = ( / 3 ) | ϕ . Es erwirbt die Phase e ich 2 π / 3 auf 2 π Drehung, und es ist invariant unter 6 π Drehung. Warum so | ϕ existiert nicht, während | ψ ein gültiger Eigenzustand von ist J ^ z ?
Das ist, weil J z gebunden ist J . Und die zulässigen Werte für J sind ganzzahlige Vielfache von 2 und der Grund dafür ist in physical.stackexchange.com/questions/174018/… zu sehen.
Vielen Dank für Ihre Antwort und Ihren Kommentar. Ich denke jedoch, dass meine Frage nicht beantwortet ist. Was macht J in Ihrer Antwort und Ihrem Kommentar bedeuten? Ist es ein Operatorvektor oder ein Scaler? Lassen Sie mich interpretieren, dass Sie sagen wollen, dass der Eigenwert von J ^ z ist durch ein ganzzahliges Vielfaches von beschränkt / 2 . Ich kann diese Obergrenze annehmen / 2 . Deutlich, / 3 unterhalb dieser Grenze liegt. (Vielleicht möchten Sie sagen, die untere Grenze ist / 2 , Und / 3 liegt über dieser Grenze.) Außerdem verstehe ich nicht, wo genau in dem Link, auf den Sie sich beziehen.

Kurz gesagt, jede Leiter von Eigenwerten von J ^ z , andere als diese J , ( J + 1 ) , , J Wo J ganzzahlig oder halbzahlig ist, nicht mindestens in einer Richtung enden würde und daher den im Fragetext gezeigten endlichen Schranken widersprechen würde.

Um genauer zu sein, wählen Sie einen beliebigen Eigenzustand aus | X a , Y β , Wo Y β wird nicht als ganzzahliges oder halbzahliges Vielfaches von angenommen . Durch wiederholtes Anwenden des Erhöhungsoperators darauf klettern wir die Leiter hinauf als

J ^ + k | X a , Y β = | X a , Y β + k C + ( X a , Y β + ( k 1 ) ) C + ( X a , Y β )
für k = 1 , 2 , . Da es die obere Schranke für den Eigenwert von gibt J ^ z , es muss eine ganze Zahl sein K 0 so dass
C + ( X a , Y β + ( k 1 ) ) 0 , k = 1 , , K , C + ( X a , Y β + K ) = 0.
Schreiben wir den größten Eigenwert auf dieser Leiter als
μ := Y β + ( K 1 ) ,
und der zugehörige Eigenzustand als
| v 0 := | X a , μ .
Von hier aus klettern wir die Leiter hinunter und sehen eine Bedingung, dass die Leiter bei endlichen Stufen abschneidet.

Wir schreiben

(1) | v k := J ^ k | v 0 = | X a , μ k C ( X a , μ ( k 1 ) ) C ( X a , μ ) ,
für k = 1 , 2 , . Dann gilt aus der Kommutierungsrelation [ J ^ + , J ^ ] = 2 J ^ z , dh,
(2) J ^ + J ^ = J ^ J ^ + + 2 J ^ z ,
daraus folgt z k = 1 , 2 , ,
(3) J ^ + | v k = | v k 1 k [ 2 μ ( k 1 ) ] .
Um diese Formel zu überprüfen, können wir die überprüfen k = 1 Fall durch Anwendung beider Seiten von Gl. ( 2 ) An | v 1 , und dann können wir die mathematische Induktion für verwenden k > 1 .

Für die Leiter von μ k ( k = 1 , 2 , ), die in Gl. ( 1 ), um bei einem endlichen Wert abzuschneiden, muss eine Ganzzahl vorhanden sein K ' 0 so dass

C ( X a , Y β ( k 1 ) ) 0 , k = 1 , , K ' , C ( X a , Y β K ' ) = 0.
Das bedeutet, dass
| v K ' 0 , | v K ' + 1 = 0.
Durch Anwendung der linken Seite von Gl. ( 2 ) An | v K ' ,
J ^ + J ^ | v K ' = J ^ + | v K ' + 1 = 0.
Durch Anwendung der rechten Seite von Gl. ( 2 ) An | v K ' und mit Gl. ( 3 ),
[ J ^ J ^ + + 2 J z ] | v K ' = J ^ | v K ' 1 K ' [ 2 μ ( K ' 1 ) ] + | v K ' 2 [ μ K ' ] = | v K ' ( K ' + 1 ) [ 2 μ K ' ] .
Die Ergebnisse der beiden obigen Gleichungen müssen gleich sein, und da K ' + 1 > 0 , es folgt dem
μ = K ' 2 .
Daher sehen wir, dass der größte Eigenwert μ auf der Leiter muss ein ganzzahliges oder halbzahliges Vielfaches von gewesen sein , oder andernfalls würde die Leiter unendlich weiter nach unten gehen und der Existenz der unteren Grenze widersprechen.

Als Ergebnis werden die Eigenwerte von J ^ z muss sein

K ' 2 K ' , K ' 2 ( K ' 1 ) , , K ' 2 ,
für eine ganze Zahl K ' 0 .

Ich habe dieses Argument aus dem Buch genommen,

  • BC Hall, Quantentheorie für Mathematiker (Springer, 2013).

Siehe Beweis von Satz 17.4 darin. Gl. (17.12) des Buches entspricht Gl. ( 3 ) Hier. [ v J auf der rechten Seite von Gl. (17.12) sollte lauten v J 1 .]